You are on page 1of 15

Vectors

(c)

45

3
45
8.

Fundamentals of Vectors
1.

on yThe vector projection of a vector 3


i 4k
axis is

2.

(b) 4

(c) 3

(d) Zero

4.

5.

5
45

The vector that must be added to the vector


and 3
so that the

i 3j 2k
i 6j 7k

9.

(a) 4
i 2j 5k

(b) 4
i 2j 5k

(c) 3
i 4j 5k

(d) Null vector

(a) 3
i 5j 2k

(a) 2

(b) 3

(c) 4

(d) 5

(b) 3
i 2j 5k

(d) None of these

(a)
i
j 10k

(b)
i j 5k

A hall has the dimensions 10m 12m 14 m. A


fly starting at one corner ends up at a
diametrically opposite corner. What is the
magnitude of its displacement

(c)
i j

(d) 2
i 4j 6k

(a) 17 m

(b) 26 m

(c) 36 m

(d) 20 m

If a particle moves from point P (2,3,5) to point Q


(3,4,5). Its displacement vector be

A force of 5 N acts on a particle along a direction


making an angle of 60 with vertical. Its vertical
component be

10.

(a) 10 N

(b) 3 N

100 coplanar forces each equal to 10 N act on a


body. Each force makes angle / 50 with the
preceding force. What is the resultant of the forces

(c) 4 N

(d) 2.5 N

(a) 1000 N

(b) 500 N

(c) 250 N

(d) Zero

If A 3
i 4j and B 7
i 24
j, the vector
having the same magnitude as B and parallel to A
is

(c)

20
i 15
j

(c)

(b) 5 2

(c) 4

(d) 2 10

13.

14.

(a)

1
45

45
,

2
45

45

and

and

3
45

45

(b) Null vector


2 (d)

Given vector A 2
i 3
j, the angle between A
and y-axis is
1

is a

[CPMT 1993]

vector A are

1
1
i
j
2
2

(c) Vector of magnitude


Scalar

2
3
A

The expression
(a) Unit vector

the direction of cosines of the


If A 2
i 4
j 5k

The magnitude of a given vector with end points


(4, 4, 0) and ( 2, 2, 0) must be
(a) 6

(d)

3A

12.

(d) 15
i 20
j

(b)

11.

(b) 15
i 10
j

Vector A makes equal angles with x, y and z axis.


Value of its components (in terms of magnitude of
A ) will be
(a)

7.

(d)

How many minimum number of coplanar vectors


having different magnitudes can be added to give
zero resultant

(a) 5
i 20
j

6.

45

and

45

Position of a particle in a rectangular-co-ordinate


system is (3, 2, 5). Then its position vector will be

(c) 5
i 3j 2k

3.

resultant vector is a unit vector along the y-axis is

[RPMT 2004]

(a) 5

, 0 and

(a) tan

3/ 2

(c) sin1 2 / 3

(b)
15.

(b) tan

(d) cos1 2 / 3

The unit vector along


i
j is

(a) k
(c)

(b)
i
j

i j
2

2/ 3

(d)

i j
2

Vectors
16.

. Its length
A vector is represented by 3
i
j 2k

in XY plane is

25.

(c)
17.

18.

10

[EAMCET (Engg.)

(b)

14

(a) 1

(d)

(c)

Five equal forces of 10 N each are applied at one


point and all are lying in one plane. If the angles
between them are equal, the resultant force will
be
[CBSE PMT 1995]
(a) Zero

(b) 10 N

(c) 20 N

(d) 10 2N

26.

(b) 45

(c) 22.5

(d) 30

27.

23.

Any vector in an arbitrary direction can always be


replaced by two (or three)

0.11

(d)

0.39

The unit vector parallel to the resultant of the

(a)

1
)
(3i 6j 2k
7

(b)

1
)
(3i 6j 2k
7

(c)

1
)
(3i 6j 2k
49

(d)

1
)
(3i 6j 2k
49

Surface area is

[J&K CET 2002]

(a) Scalar

(d) It is not possible to resolve a vector

(c) Neither scalar nor vector (d)


vector

Angular momentum is

[MNR 1986]

(a) A scalar

(b) A polar vector

(c) An axial vector

(d) None of these

29.

(a) Pressure

(b) Surface tension

(c) Moment of inertia

(d) None of these

QP

(c) PQ

Q
(d) P Q P

The
position
vector
of
a
particle
is

r (a cost)
i (a sint)
j . The velocity of the
particle is
[CBSE PMT 1995]

(c) r
30.

(c) Directed towards the origin


[AFMC

1998]

(a) Displacement

(b) Electric field

(c) Acceleration

(d) Work

The

1
(i j k)
3

angle

between

(b) r

1
(i j k)
2

(d) r

1
(i j k)
2

the

two

and B 3
is
A 3
i 4j 5k
i 4
j 5k

vectors

[DPMT 2000]

(d) Directed away from the origin


Which of the following is a scalar quantity

1
(i j k)
3

(a) r

(b) Perpendicular to the position vector

c k

(Engg.) 2003]

With respect to a rectangular cartesian coordinate


system, three vectors are expressed as

Z-axis respectively. The unit vectors r


along the
direction of sum of these vector is
[Kerala CET

(b) | P| | Q|

Both scalar and

are unit vectors, along the X, Y and


where
i,
j, k

If P Q then which of the following is NOT


correct

Q
(a) P

(b) Vector

a 4
i
j , b 3
i 2
j and

Which of the following is a vector

[CBSE

A boy walks uniformally along the sides of a


rectangular park of size 400 m 300 m, starting
from one corner to the other corner diagonally
opposite. Which of the following statement is
incorrect
[HP PMT 1999]

(c) Arbitrary vectors which have the original


vector as their resultant

(a) Parallel to the position vector

24.

0.01

(b)

by

and B

vectors A 4
i 3
j 6k
i 3
j 8k
is [EAMCET 2000]

28.

22.

represented

(d) His velocity is not uniform throughout the walk

(b) Mutually perpendicular vectors which have


the original vector as their resultant

21.

is

(c) His displacement is 500 m

(a) Parallel vectors which have the original vector


as their resultant

20.

vector

(b) His displacement is 700 m

The angle made by the vector A


i j with xaxis is
(a) 90

unit

(a) He has travelled a distance of 700 m

[EAMCET (Engg.) 1999]

19.

, then the value of c is


0.5
i 0.8
j ck
PMT 1999; EAMCET 1994]

1994]

(a) 2

If

31.

(a) 60

(b) Zero

(c) 90

(d) None of these

The position vector of a particle is determined by

the expression r 3t2


i 4t2j 7k

Vectors
The distance traversed in first 10 sec is

[DPMT

2002]

32.

33.

(a) 500 m

(b) 300 m

(c) 150 m

(d) 100 m

A 4
i 3
j and B 8
i 8
j will be

24
i 5
j
13

(b)

(c)

6
i 5j
13

(d) None of these

12
i 5
j
13

The

between

7.

two

vectors

and B 3
will be
A 3
i 4j 5k
i 4
j 5k
[Pb. CET 2001]

9.

(a) 90

(b) 0

(c) 60

(d) 45

Addition and Subtraction of Vectors


1.

There are two force vectors, one of 5 N and other


of 12 N at what angle the two vectors be added to
get resultant vector of 17 N, 7 N and 13 N
respectively
(a) 0, 180 and 90
(b) 0, 90 and 180
(c) 0, 90 and 90

2.

If

3.

10.

A B

(b) 5 5, tan1 (1 / 2)

(c) 10, tan1 (5)

(d) 25, tan1 (3 / 4)

(a) 40 m/s NW

(b) 20 2 m/s NW

(c) 40 m/s SW

(d) 20 2 m/s SW

If the sum of two unit vectors is a unit vector, then


magnitude of difference is
[CPMT 1995; CBSE

(c) 1 /

Two forces, each of magnitude F have a resultant


of the same magnitude F. The angle between the
two forces is
(a) 45

(b) 120

(c) 150

(d) 60

For the resultant of the two vectors to be


maximum, what must be the angle between them
(a) 0

(b) 60

(c) 90

(d) 180

A particle is simultaneously acted by two forces


equal to 4 N and 3 N. The net force on the particle
is [CPMT 1979]
(a) 7 N

(b) 5 N

(c) 1 N
N

(d) Between 1 N and 7

Two vectors

C lies

(d)

B lie in a plane, another

outside

this

plane,

then

the

11.

If the resultant of the two forces has a magnitude


smaller than the magnitude of larger force, the
two forces must be
(a) Different both in magnitude and direction
(b) Mutually perpendicular to one another
(c) Possess extremely small magnitude
(d) Point in opposite directions

12.

Forces F1 and F2 act on a point mass in two


mutually perpendicular directions. The resultant
force on the point mass will be
[CPMT 1991]

(a) F1 F2

(b)

A and

(c) Lies in the plane containing A B

(d) Lies in the plane containing C

A truck travelling due north at 20 m/s turns west


and travels at the same speed. The change in its
velocity be

v2

(a) Can be zero


(b) Cannot be zero

will be

(a) 5, tan1 (3 / 4)

(a)

[CPMT 1983]

(d) 180, 0 and 90

PMT 1989]

v1

resultant of these three vectors i.e., A B C

[UPSEAT 1999]

4.

An object of m kg with speed of v m/s strikes a


wall at an angle and rebounds at the same speed
and same angle. The magnitude of the change in
momentum of the object will be

vector

A 4
i 3
j and B 6
i 8
j then magnitude

and direction of

(d) 5
i 4
j 10k

(d) 2 mv

8.

the

(c) 4
i 5
j 20k

[CBSE PMT 1990]

(d) 5

angle

(b) 20
i 5
j 4k

(c) 0

(b) 10 2

(a) 20
i 5
j 4k

(b) 2 mv sin

[KCET 1997]

.
C 6
i 2k

(a) 2mv cos

The component of vector A 2


i 3
j along the

(c) 5 2
34.

6.

[BHU 1995]

(a)

vector
i
j is

and
A 2
i
j, B 3
jk

Value of A 2B 3C would be

Unit vector parallel to the resultant of vectors

(a)

5.

(c)

F12 F22

(b) F1 F2
(d) F12 F22

Vectors
13.

If

and B is
(a) 60
(c) 120
14.

the angle between

| A B | | A | | B |,

(c)

Let

the angle between two nonzero vectors A


and B be 120 and resultant be C

22.

| A B|

(d) C may be equal to | A B |


The magnitude of vector A, B

and

23.
C

are

(a) 0
16.

(b)

(c) / 2
(d) / 4
Magnitude of vector which comes on addition of

24.

(c)
17.

(b)

13.2

202

(d)

160

25.

A particle has displacement of 12 m towards east


and 5 m towards north then 6 m vertically upward.
The sum of these displacements is

18.

(b) 10.04 m
(d) None of these
three
vectors

, B

A 3
i 2
jk
i 3
j 5k

and

26.

19.

(b)
(d)
27.

5
12

(b) cos

12
13

(d) cos

5
13

7
13

The resultant of two vectors A and B is


perpendicular to the vector A and its magnitude is
equal to half the magnitude of vector B. The angle
between A and B is
(a) 120
(b) 150
(c) 135
(d) None of these
What vector must be added to the two vectors
may be a unit vector along

(b) B C A

x-axis

[BHU 1990]

(c) C A B

(a) 2
i
jk

(b) 2
i
jk

(d) A B C 0

(c) 2
i
jk

(d) 2
i
jk

Let C A B then

28.

(a) | C | is always greater then | A |


(b) It

is

possible

to

have

| C | | A |

and

(c) C is always equal to A + B


The value of the sum of two vectors A and B
with as the angle between them is

(a) Zero

(b) tan1(P / Q)

(c) tan1(Q / P )

(d)

tan1(P Q) /(P Q)

(d) C is never equal to A + B

[BHU 1996]

What is the angle between P and the resultant


of (P Q) and (P Q)

| C | | B |

21.

and 2
, so that the resultant

i 2
j 2k
i
jk

(a) A B C

20.

(b) 8 N and 8 N
(d) 2 N and 14 N

If vectors P, Q and R have magnitude 5, 12 and 13


units and P Q R, the angle between Q and R
is [CEET 1998]

(c) cos

form
C 2
i
j 4k

(a) An equilateral triangle


Isosceles triangle
(c) A right angled triangle
No triangle
For the figure

The sum of two forces acting at a point is 16 N. If


the resultant force is 8 N and its direction is
perpendicular to minimum force then the forces
are
[CPMT 1997]

(a) cos

[AIIMS 1998]

(a) 12
(c) 14.31 m
The

(b) 10, 10, 20


(d) 10, 20, 40

When three forces of 50 N, 30 N and 15 N act on a


body, then the body is

(a) 6 N and 10 N
(c) 4 N and 12 N

[BHU 2000]

136

Following sets of three forces act on a body. Whose


resultant cannot be zero
[CPMT 1985]

(c) In equilibrium
(d) Moving with an acceleration

two vectors, 6
i 7
j and 3
i 4
j is
(a)

(d)

(a) At rest
(b) Moving with a uniform velocity

respectively 12, 5 and 13 units and A B C


then the angle between A and B is
[CPMT 1997]

A 2 B 2 2AB sin

(a) 10, 10, 10


(c) 10, 20, 23

| A B|

(c) C must be greater than

(b)

A 2 B 2 2AB sin

| A B|

(b) C must be less than

A 2 B 2 2AB cos

A 2 B 2 2AB cos

(b) 0
(d) 90

(a) C must be equal to

15.

(a)

29.

The resultant of P and Q is perpendicular to


P . What is the angle between P and Q
(a) cos1(P / Q)

(b) cos1( P / Q)

Vectors
(c) sin1 (P / Q)
30.

31.

32.

(c) 20 km/hour

(d) sin1 ( P / Q)

Maximum and minimum magnitudes


of the
resultant of two vectors of magnitudes P and Q are
in the ratio 3 : 1. Which of the following relations
is true
(a) P 2Q

(b) P Q

(c) PQ 1

(d) None of these

(c) cos1(1 / 4)
33.

and C are 3, 4

and 5 units respectively. If A B C , the angle

between A and B is

(b) cos1(0.6)

2
7

1
(c) tan

39.

F2 are acting on a body.

(b) cos1(1 / 2)
40.

(d) cos1(1 / 4)

Given that A B C and that C is to A .


Further if | A | | C |, then what is the angle

(d)

While travelling from one station to another, a car


travels 75 km North, 60 km North-east and 20 km
East. The minimum distance between the two
stations is
[AFMC 1993]
(a) 72 km

(b) 112 km

(c) 132 km

(d) 155 km

A scooter going due east at 10 ms1 turns right


through an angle of 90. If the speed of the
scooter remains unchanged in taking turn, the
change is the velocity of the scooter is
[BHU 1994]

between A and B
(a)
(c)
34.

radian

(b)

3
radian
4

(a) 20.0 ms south eastern direction

radian

(b) Zero
(c) 10.0 ms1 in southern direction

(d) radian

(d) 14.14 ms1 in south-west direction


41.

(c) 4
i 6
j

(b) 4
i 6
j

(a) 22.36 km

(d) 4
i 6
j

(c) 5 km

A plane is revolving around the earth with a speed


of 100 km/hr at a constant height from the surface
of earth. The change in the velocity as it travels
half circle is

42.

(a) 200 km/hr

(b) 150 km/hr

(c) 100 2 km/ hr

(d) 0

What

displacement

displacement

be

added

to

to

give

Two
forces

act on a single point.


F2 10
i 5
j 15k

[AMU 1995]

the
a

(d) 20 km

and
F1 5
i 10
j 20k

(b) 45

(c) 60

(d) 90

Which pair of the following forces will never give


resultant force of 2 N
[HP PMT 1999]

(a) 18
i 6
j

(a) 2 N and 2 N

(b) 1 N and 1 N

(c) 1 N and 3 N

(d) 1 N and 4 N

18
i 6
j

(b)

32
i 13
j

(d) 25
i 13
j

A body moves due East with velocity 20 km/hour


and then due North with velocity 15 km/hour. The
resultant velocity
[AFMC 1995]

(a) 5 km/hour

43.

(a) 30

displacement of 7.0 m pointing in the x- direction


(c)
37.

must

25
i 6
j m

(b) 2 km

The angle between F1 and F2 is nearly

[RPET 1998; KCET 2000]

36.

A person goes 10 km north and 20 km east. What


will be displacement from initial point
[AFMC 1994, 2003]

[AMU 1996]

(a) 4
i 6
j
35.

A body is at rest under the action of three forces,

two of which are F1 4


i , F2 6j, the third
force is


A, B

[CBSE PMT 1990]

One force is double that of the other force and the


resultant is equal to the greater force. Then the
angle between the two forces is
(a) cos1(1 / 2)

The magnitudes of vectors

(a)

The resultant of two vectors P and Q is R. If


Q is doubled, the new resultant is perpendicular
to P. Then R equals
(a) P
(b) (P+Q)
(c) Q
(d) (PQ)
Two forces, F1 and

38.

(d) 25 km/hour

(b) 15 km/hour

44.

Two forces 3N and 2 N are at an angle such that


the resultant is R. The first force is now increased
to 6N and the resultant become 2R. The value of
is [HP PMT 2000]
(a) 30

(b) 60

Vectors
(c) 90
45.

Three
are in
forces
forces

(d) 120

(c) 238

concurrent forces of the same magnitude


equilibrium. What is the angle between the
? Also name the triangle formed by the
as sides

53.

Two vectors

A B A B . Then
(a)

(b) 120 equilateral triangle

If

and B will be

1.

, then angle between A

48.

(a) 90

(b) 120

(c) 0

(d) 60

The maximum and minimum magnitude of the


resultant of two given vectors are 17 units and 7
unit respectively. If these two vectors are at right
angles to each other, the magnitude of their
resultant is
[Kerala CET (Engg.) 2000]

(a) 1
(c)
2.

is

1
2

1
2

(d) 1

and 4

If two vectors 2
i 3j k
i 6
j k

(b) 16

(a) 0

(b) 2

(c) 18

(d) 13

(c) 3

(d) 4

The vector sum of two forces is perpendicular to


their vector differences. In that case, the forces

3.

(a) Are equal to each other in magnitude

A body, acted upon by a force of 50 N is displaced


through a distance 10 meter in a direction making
an angle of 60 with the force. The work done by
the force be

(b) Are not equal to each other in magnitude

(a) 200 J

(b) 100 J

(c) 300

(d) 250 J

y component of velocity is 20 and x component of


velocity is 10. The direction of motion of the body
with the horizontal at this instant is

4.

(a) tan1(2)

(b) tan1(1 / 2)

(c) 45

(d) 0

) N. If the displacement in meters


(4
i
j 3k

then work done will be


[CMEET 1995; Pb. PMT 2002, 03]

Two forces of 12 N and 8 N act upon a body. The


resultant force on the body has maximum value of
[Manipal 2003]

(a) 4 N

(b) 0 N

(c) 20 N

(d) 8 N

5.

(a) P / 2
(c)

(b) P / 4
(d) 2P

(a) 100 J

(b) 200 J

(c) 300 J

(d) 250 J

If for two vector

6.

(b) 2

(c) 3

(d) None of these

The angle between the vectors A and B is .


The value of the triple product A .(B A ) is
[CBSE PMT 1991, 2005]

(a) A 2 B

[BHU 2000]

(c)

274

(b) 38

and B , sum (A B) is

(a) 1

The vectors 5i 8 j and 2i 7 j are added. The


magnitude of the sum of these vector is
(a)

perpendicular to the difference (A B) . The ratio


of their magnitude is

Two equal forces (P each) act at a point inclined to


each other at an angle of 120. The magnitude of
their resultant is
[Karnataka CET
2004]

to
A particle moves from position 3
i 2
j 6k
due to a uniform force of
14
i 13
j 9k

[Manipal 2003]

52.

(b)

(a) 14

(d) Are equal to each other

51.

are parallel to each other then value of be

(c) Cannot be predicted

50.

is perpendicular to the
If a vector 2
i 3
j 8k
[CBSE PMT 2005]

[CBSE PMT 2003]

49.

(d) B 0

. Then the value of


vector 4j 4
i k

[CBSE PMT 2001]

47.

that

Multiplication of Vectors

(c) 120, 30, 30 an isosceles triangle


46.

such


(b) A B 0


A. B 0

(c) A 0

(a) 60 equilateral triangle

| A B| | A| | B|

are

[AMU (Med.) 2000]

[JIPMER 2000]

(d) 120 an obtuse angled triangle

(d) 560

A and B

A 2 B sin

(b) Zero
(d) A 2 B cos

Vectors
7.

If

A B B A then the angle between A

15.

If | V1 V 2 | | V1 V 2 | and V2 is finite, then


[CPMT 1989]

and B is

(a) V1 is parallel to V2

[AIEEE 2004]

8.

(a) / 2

(b) / 3

(b) V 1 V 2

(c)

(d) / 4

(c)

If

A 3
i j 2k

B 2
i 2j 4k

and

9.

(d) | V 1 | | V 2 |

then

value of | A B | will be

16.

(a) 8 2

(b) 8 3

(c) 8 5

(d) 5 8

) m about the
acting at the point r (3
i 2
j 3k

origin be

17.

(b)

17
i 6
j 13k

18.

(d)
(c) 6
i 6
j 12k

If
A B C, then
statements is wrong
(a)

(b)

C A

(c) C (A B)
11.

12.

which

of

the

C B

19.

(d) C (A B)

(d) +11 J

and
The angle between two vectors 2
i 3j k
[EAMCET 1990]

(a) 0

(b) 90

(c) 180

(d) None of the above


(
i
j)

The angle between the vectors

(a) 30

(b) 45

(c) 60

(d) 90

particle

moves

with

m / s under
6
i 4
j 3k

If a particle of mass m is moving with constant


velocity v parallel to x-axis in x-y plane as shown
in fig. Its angular momentum with respect to origin
at any time t will be

the

a
influence

(b) 45 J/s

i
(c) mvb

i
(d) mv

(c) 25 J/s

(d) 195 J/s

vectors

F 1 2i 5k

and

(a) 20

(b) 23

(c) 5 33

(d) 26

Consider a vector

20.

If

P .Q PQ,

then angle between P and Q is

21.

(a) 0

(b) 30

(c) 45

(d) 60

A force

F 5
i 6j 4k

acting on a body,

. Work done
produces a displacement S 6
i 5k
by the force is
[KCET 1999]

that is perpendicular to F is

(a) 10 units

(b) 18 units

(a) 4
i 3
j

(b) 6
i

(c) 11 units

(d) 5 units

(c) 7k

(d) 3
i 4
j

22.

The angle between the two vectors A 5


i 5j

Two vectors A and B are at right angles to


each other, when
[AIIMS 1987]

and B 5
i 5
j will be
(a) Zero

(b) 45

(a) A B 0

(b) A B 0

(c) 90

(d) 180

(c)

(d)

A B 0

[AIIMS 1999]

F 4
i 3
j. Another vector

A. B 0

of

[CBSE PMT 2000]

(a) 35 J/s

two

velocity

instantaneous power applied to the particle is

(b) mvbk

Consider

and

N. The
constant force F 20
i 15
j 5k

(a) mvbk

[MP PMT 1987]

14.

(b) +13 J

(c) +7 J

[EAMCET 1995]

following

. The magnitude of the scalar


F 2 3j 4k
product of these vectors is

13.

(a) 7 J

) is
(
jk

17
i 6
j 13k

10.

[MP PMT 1995]

is

i 2
j 4k

[CBSE PMT 1995]

(a) 6
i 6
j 12k

A force F (5
i 3
j) Newton is applied over a
particle which displaces it from its origin to the
point r (2
i 1
j) metres. The work done on
the particle is

)N
F (2
i 3
j 4k

The torque of the force

V1 and V2 are mutually perpendicular

[CPMT 2000]

Vectors
23.

The

P a
i a
j 3k

vector

and

(a) 0

are perpendicular to each other.


Q a
i 2j k

The positive value of a is

(c)

[AFMC 2000; AIIMS 2002]

24.

(a) 3

(b) 4

(c) 9

(d) 13

31.

A body, constrained to move in the Y-direction is


subjected
to
a
force
given
by
) N. What is the work
F (2
i 15
j 6k

done by this force in moving the body a distance


10 m along the Y-axis
[CBSE PMT 1994]

25.

32.

Let

A
i A cos
jA sin

anytime

[MNR 1991;

(a) 0

(b) 30

(c) 90

(d) 180

(b) 7.5 units

(c) 10 units

(d) 5 units

A vector

33.

The

angle

34.

)
(c) (
jk

(d) (4
i)

If for two vectors


vectors

and

F 2 is zero

B, A B 0,

the

35.

36.

(d) Act at an angle of 30


vectors

(A B)

37.

(a) Zero

(b)

(c) / 4

(d) / 2

is

(P Q)

the

angle

1
3
2

A vector

1
(b) cos

(d) sin1

5
3
5
3

points vertically upward and

(a) Zero

(b) Along west

(c) Along east

(d) Vertically downward

) is
Angle between the vectors (
i
j) and (
jk

(a) 90

(b) 0

(c) 180

(d) 60

The position vectors of points A, B, C and D are

(c) Antiparallel
(d) Inclined at an angle of 60

and

(B A) is

What

by

(a) Perpendicular
(b) Parallel

(c) Act at an angle of 60

between

given

and D 4
i 6
j then the displacement vectors
AB and CD are

(b) Are parallel to each other

angle

vectors

, B 4
, C 7

A 3
i 4
j 5k
i 5
j 6k
i 9
j 3k

(a) Are perpendicular to each other

The

two

points towards north. The vector product A B


is
[UPSEAT 2000]

then F 2 could be
(b) (
i
j)

between

1
(c) sin

F 1 is along the positive X-axis. If its

(a) 4
j

(d)

6i 6 j 3k and 7i 4 j 4k is
[EAMCET (Engg.) 1999]
1
(a) cos

[MP PMT 1987]

vector.

i B cos j B sin

The area of the parallelogram represented by the


(a) 14 units

any

(b)

i B sin j B cos
(c)

is

between

be

i B sin j B cos

A particle moves in the x-y plane under the action


of a force F such that the value of its linear

vector product with another vector

30.

(d) 0

i B cos j B sin
(a)

vectors A 2
i 3
j and B
i 4j is

29.

(c) 1

Another vector B which is normal to A is

UPSEAT 2000]

28.

(b) 3

(d) 190 J

F and P at a given time t. will be

27.

(a) 6

(c) 160 J

at

The resultant of the two vectors having magnitude


2 and 3 is 1. What is their cross product

[BHU 1997]

(P )

(d)

(b) 150 J

Px 2 cost, py 2 sint. The angle

26.

(a) 20 J

momentum

(b)

between

If

force

(F ) 4
i 5
j and

then the work done is


(s) 3
i 6k
[Manipal 1995]

( P Q)

and

4 3
(c) 6 3
(a)

5 6
(d) 4 6
(b)

displacement

Vectors
38.

If | A B | | A . B |,

then angle between

(b) y-axis

and B will be

(c) z-axis
[AIIMS 2000; Manipal 2000]

39.

40.

(d) Line at equal angles to all the three axes

(a) 30

(b) 45

(c) 60
In an clockwise system

(d) 90

(a) j k
i

i .
i 0
(b)

(a)

(c) j j 1

.
(d) k
j 1

(c) 3A 3B

46.
[CPMT 1990]

The linear velocity of a rotating body is given by


v r, where is the angular velocity and

47.

41.

31 units

(c)

37 units

(d)

41 units

Three vectors

42.

[AIIMS 1996]

(b) c

(c)

(d) b c

The diagonals of a parallelogram are

2
i

43.

14
i 34
j 16k

48.

and

. What is the area of the parallelogram


2j

(a) 0.5 units

(d)

The value of (A B) (A B) is
[RPET 1991, 2002; BHU 2002]

(a) b
b. c

21
i 4
j 4k

(c)

and c satisfy the relation


a. c 0. The vector
a is

and

a. b 0

acting
Find the torque of a force F 3
i
j 5k

(b) 4
i 4
j 6k

a, b

parallel to

(d) All of these

(a) 14
i 38
j 16k

, then | v| is
r 4
j 3k

(b)

(b) A B

[CPMT 1997; CBSE PMT 1997; CET 1998; DPMT 2004]

and the radius vector


body is
i 2j 2k

29 units

A B

at the point r 7
i 3j k

r is the radius vector. The angular velocity of a

(a)

Two vector A and B have equal magnitudes. Then


the vector A + B is perpendicular to

49.

(b) 1 unit

(c) 2 units
(d) 4 units
What is the unit vector perpendicular to the

(a) 0

(b) A 2 B 2

(c) B A

(d) 2(B A)

If
and B are perpendicular vectors and

and B 2
.
vector A 5
i 7
j 3k
i 2
j ak

The value of a is

and 6

following vectors 2
i 2
jk
i 3
j 2k

(a)

i 10
j 18k

[EAMCET 1991]

(b)

5 17

i 10
j 18k

50.

5 17
(c)

17

the

vectors

j 3k

and

51.

(a)

61 sq.unit

(b)

59 sq.unit

(c)

49 sq.unit

(d)

52 sq.unit

The

position

of

(a) x-axis

and

accelerates

with

particle

is

given

20

(a) 10 2

(b)

(c) 2 10

(d) 10

Two

adjacent

sides

of

parallelogram

are

. What is the area of parallelogram


3
i 2
jk
[AMU 1997]

by

r (i 2 j k) momentum P (3i 4 j 2k).


The angular momentum is perpendicular to
[EAMCET (Engg.) 1998]

F 6
i 8
j 10k

and
represented by the two vectors
i 2
j 3k

is

i 2
jk

45.

A force vector applied on a mass is represented as

[CMEET 1995]

The area of the parallelogram whose sides are


by

(d) 8

1 m/s . What will be the mass of the body in kg.

5 17
represented

(c) 7

(d)

i 10
j 18k
44.

(b) 8

i 10
j 18k
5

(a) 2

52.

(a) 8

(b) 8 3

(c) 3 8

(d) 192

and
The position vectors of radius are 2
i j k
while those of linear momentum are
2
i 3
jk

Vectors

53.

[Pb. CET 1996]

. Then the angular momentum is


2
i 3
jk
[BHU 1997]

(a) A

(b) A

(a) 2
i 4k

(b) 4
i 8k

(c) Zero vector

(d) Zero

(c) 2
i 4
j 2k

(d) 4
i 8k

What

is

the

value

of

linear

Lami's Theorem
velocity,

and r 5

3
i 4j k
i 6
j 6k

if

[CBSE

1.

PMT 1999; CPMT 1999, 2001;

wt, sin1 0.9659, the value of R is ( in kg wt)

Pb. PMT 2000; Pb. CET 2000]

(a) 6
i 2
j 3k

(b) 6
i 2
j 8k

(c) 4
i 13
j 6k

(d)

[CET 1998]

(a) 0.9659
(b) 2

18
i 13
j 2k

54.

[Haryana CEET 2002]

(b) 1

(c)
55.

When

(d)

(d) None of these



A.B | A || B |,

2.

then

56.

and
and
and
and

If | A B|

are perpendicular to each other

(a)

act in the opposite direction


can act in any direction

3 A.B, then the value of| A B|

[CBSE PMT 2004]

AB
(a) A 2 B 2

1/ 2

(b)

3 AB)1 / 2

A B

3.

(d)

acting on a particle
F 3
i cj 2k

in its
causes a displacement S 4
i 2
j 3k

A force

58.

(b) 6

(c) 1

(d) 0

A force F (5
i 3
j) N is applied over a particle

which displaces it from its original position to the point

s (2
i 1
j) m. The work done on the particle is
[BHU 2001]

(a) + 11 J
(c) + 13 J
59.

(b) + 7 J

(d) 7 J

If a vector A is parallel to another vector B

then the resultant of the vector A B will be


equal to

(c)

P
Q
R

tan
tan
tan

(d)

P
Q
R

sin
sin
sin

If a body is in equilibrium under a set of noncollinear forces, then the minimum number of
forces has to be
[AIIMS 2000]

4.

own direction. If the work done is 6J, then the


value of c will be
[DPMT 1997]
(a) 12

P
Q
R

sin
sin
sin

(a) Four
(c) Two

1/ 2

(A B AB)
57.

1
2

P
Q
R

(b)
cos
cos
cos

is

A body is in equilibrium under the action of three


coplanar forces P, Q and R as shown in the figure.
Select the correct statement

act in the same direction

(c) (A 2 B 2

[AFMC 1994]

[Orissa JEE 2003]

(a) A

(b) A

(c) A

(d) A

150o

(c) 1

Dot product of two mutual perpendicular vector is


(a) 0

P, Q and R are three coplanar forces acting at a


point and are in equilibrium. Given P = 1.9318 kg

5.

(b) Three
(d) Five

How many minimum number of non-zero vectors


in different planes can be added to give zero
resultant
(a) 2
(b) 3
(c) 4
(d) 5
As shown in figure the tension in the horizontal
cord is 30 N. The weight W and tension in the
string OA in Newton are
[DPMT 1992]

(a) 30

3, 30

(b) 30

3, 60

(c) 60

3, 30

(d) None of these

Relative Velocity

30o
30 N
O
W

Vectors
1.

2.

3.

Two cars are moving in the same direction with the


same speed 30 km/hr. They are separated by a
distance of 5 km, the speed of a car moving in the
opposite direction if it meets these two cars at an
interval of 4 minutes, will be
(a) 40 km/hr

(b) 45 km/hr

(c) 30 km/hr

(d) 15 km/hr

(a) 10 km/hr

(b) 20 km/hr

(c) 30 km/hr

(d) 40 km/hr

(c) 10 3 km/ h

5.

9.

10.

(c) 15 km/h

(d) 145 km/h

A thief is running away on a straight road on a


jeep moving with a speed of 9 m/s. A police man
chases him on a motor cycle moving at a speed of
10 m/s. If the instantaneous separation of jeep
from the motor cycle is 100 m, how long will it
take for the policemen to catch the thief
(a) 1 second

(b) 19 second

(c) 90 second

(d) 100 second

(d) 5 / 3 km/ h

(a) If he crosses the river in minimum time

A boat is moving with a velocity 3i + 4j with respect


to ground. The water in the river is moving with a
velocity 3i 4j with respect to ground. The
relative velocity of the boat with respect to water
is
[CPMT 1998]
(a) 8j

(b) 6i 8j

(c) 6i +8j

(d) 5 2

(b) 15 s

(c) 8 s

(d) 10 s

A river is flowing from east to west at a speed of 5


m/min. A man on south bank of river, capable of
swimming 10m/min in still water, wants to swim
across the river in shortest time. He should swim

du
v

(c) For x to be minimum he has to swim in a

v
sin1
2
u
direction making an angle of

A 150 m long train is moving to north at a speed


of 10 m/s. A parrot flying towards south with a
speed of 5 m/s crosses the train. The time taken
by the parrot the cross to train would be:
(a) 30 s

du
v

(b) x can not be less than

with the direction of the flow of water


(d) x will be max. if he swims in a direction

v
sin1
2
u

making an angle
of
direction of the flow of water
11.

with

(b) Due north-east

A man sitting in a bus travelling in a direction from


west to east with a speed of 40 km/h observes
that the rain-drops are falling vertically down. To
the another man standing on ground the rain will
appear
[HP PMT 1999]

(c) Due north-east with double the speed of river

(a) To fall vertically down

[BHU 1998]

(a) Due north

(d) None of these


7.

(b) 60 km/h

(b) 5 km/h

[CBSE PMT 1992]

6.

(a) 80 km/h

A man can swim with velocity v relative to water.


He has to cross a river of width d flowing with a
velocity u (u > v). The distance through which he
is carried down stream by the river is x. Which of
the following statement is correct

In the above problem, the speed of raindrops w.r.t.


the moving man, will be
2 km/ h

A moves with 65 km/h while B is coming back of A


with 80 km/h. The relative velocity of B with
respect to A is
[AFMC 2000]

A man standing on a road hold his umbrella at 30


with the vertical to keep the rain away. He throws
the umbrella and starts running at 10 km/hr. He
finds that raindrops are hitting his head vertically,
the speed of raindrops with respect to the road will
be

(a) 10 /

4.

8.

(b) To fall at an angle going from west to east

A person aiming to reach the exactly opposite


point on the bank of a stream is swimming with a
speed of 0.5 m/s at an angle of 1200 with the
direction of flow of water. The speed of water in
the stream is
[CBSE PMT 1999]
(a) 1 m/s

(b) 0.5 m/s

(c) 0.25 m/s

(d) 0.433 m/s

(c) To fall at an angle going from east to west


(d) The information given is insufficient to decide
the direction of rain.
12.

A boat takes two hours to travel 8 km and back in


still water. If the velocity of water is 4 km/h, the
time taken for going upstream 8 km and coming
back is
[EAMCET 1990]

Vectors
(a) 2h

4.

(b) 2h 40 min
(c) 1h 20 min
(d) Cannot be estimated with the information
given
13.

A 120 m long train is moving towards west with a


speed of 10 m/s. A bird flying towards east with a
speed of 5 m/s crosses the train. The time taken
by the bird to cross the train will be

2002]

5.

14.

(b) 12 sec

(c) 10 sec

(d) 8 sec

(a) 12, 5

(b) 14, 4

(c) 5, 13

(d) 10, 8

A vector a is turned without a change in its


length through a small angle d . The value of

[Manipal 2002]

(a) 16 sec

The sum of the magnitudes of two forces acting at


point is 18 and the magnitude of their resultant is
12. If the resultant is at 90 with the force of
smaller magnitude, what are the, magnitudes of
forces
[Roorkee
1992;
AIEEE

| a| and

A boat crosses a river with a velocity of 8 km/h. If


the resulting velocity of boat is 10 km/h then the
velocity of river water is
[CPMT 2001]

6.

are respectively

(a) 0, a d

(b) a d , 0

(c) 0, 0

(d) None of these

Find the resultant of three vectors

OA, OB

and

OC shown in the following figure. Radius of the

(a) 4 km/h

(b) 6 km/h

circle is R.

(c) 8 km/h

(d) 10 km/h

(a) 2R
(b) R(1

2)

45o
45o

(c) R 2
(d) R( 2 1)
1.

2.

3.

If a vector

making angles , , and

respectively with the X, Y and Z axes respectively.

Figure shows ABCDEF as a regular hexagon. What


is the value of AB AC AD AEE AF D

Then sin2 sin2 sin2

(a)

(a) 0

(b) 1

(b) 2AO

(c) 2

(d) 3

(c) 4 AO

If the resultant of n forces of different magnitudes


acting at a point is zero, then the minimum value
of n is
[SCRA 2000]
(a) 1

(b) 2

(c) 3

(d) 4

are

same

in

same

in

(d) Yes,

same

in

vectors

magnitude making an angle of

O
A

The length of second's hand in watch is 1 cm. The


change in velocity of its tip in 15 seconds is

[IIIT 2000]

(c) Yes, when the 2 vectors are


magnitude but opposite in sense

other

8.

(c)
9.

the

(d) 6 AO

(a) Zero

(b) No

when

AO

[MP PMT 1987]

Can the resultant of 2 vectors be zero


(a) Yes, when the 2 vectors
magnitude and direction

7.

are

2
with each
3

(b)

cm/ sec
30

(d)

30 2

cm/ sec

2
cm/ sec
30

A particle moves towards east with velocity 5 m/s.


After 10 seconds its direction changes towards
north with same velocity. The average acceleration
of the particle is
[CPMT 1997; IIT-JEE 1982]

(a) Zero

(c)

1
2

(b)

m/ s2 N E

(d)

1
2

m/ s2 N W

1
2

m/ s2 S W

Vectors
10.

A force F K (y
i x
j) (where K is a positive
constant) acts on a particle moving in the x-y
plane. Starting from the origin, the particle is
taken along the positive x- axis to the point (a, 0)
and then parallel to the y-axis to the point (a, a).
The total work done by the forces F on the
particle is
[IIT-JEE 1998]

11.

(a) 2 Ka 2

(b) 2Ka 2

(c) Ka 2

(d) Ka2

(a)

(c)
(d)
(e)

If both assertion and reason are true and the


reason is the correct explanation of the assertion.
If both assertion and reason are true but reason is
not the correct explanation of the assertion.
If assertion is true but reason is false.
If the assertion and reason both are false.
If assertion is false but reason is true.

1.

Assertion

(b)

The vectors from origin to the points A and B are

A 3
i 6
j 2k

B 2
i j 2k

and

respectively. The area of the triangle OAB be


(a)

(c)
12.

5
17 sq.unit
2

(b)

3
17 sq.unit
5

(d)

2
17 sq.unit
5

(a) P W tan

[IIT 1988; CBSE PMT 1998, 2000]

14.

Reason

5.

A man crosses a 320 m wide river perpendicular to


the current in 4 minutes. If in still water he can
swim with a speed 5/3 times that of the current,
then the speed of the current, in m/min is

6.

7.

[Roorkee 1998]

(b) 40

(c) 50

(d) 60.

Assertion

Read the assertion and reason carefully to mark the


correct option out of the options given below:

Assertion

A B is perpendicular to A.B

:
:

If

| A B| | A B|

,
then

angle between A and B is 90

A B B A

:
Vector product of two vectors is
an axial vector

Reason

(b) 3 km/h
(d) 5 km/h

(a) 30

If be the angle between


A B
and B , then tan
A.B

Assertion

Reason

The speed of a boat is 5 km/h in still water. It


crosses a river of width 1 km along the shortest
possible path in 15 minutes. The velocity of the
river water is
(a) 1 km/h
(c) 4 km/h

3.

(c) T 2 P 2 W2

T P W

is 45
: i j is equally inclined to both
i
and j and the angle between
and
i

j is 90

4.

(b) T P W 0

13.

Reason

5
17 sq.unit
3

A metal sphere is hung by a string fixed to a wall.


The sphere is pushed away from the wall by a
stick. The forces acting on the sphere are shown in
the second diagram. Which of the following
statements is wrong

(d)

2.

A B is perpendicular to both

A B as well as A B.

Reason : A B as well as A B lie in the

plane containing A and B , but

A B lies perpendicular to the

plane containing A and B.


Assertion :
Angle between
i
j and
i

: If v = instantaneous velocity, r =

radius vector and = angular


velocity, then v r .
Assertion :
Minimum number of non-equal
vectors in a plane required to give
zero resultant is three.

Reason : If A B C 0, then they must lie in


one plane
Assertion :
Relative velocity of A w.r.t. B is
greater than the velocity of either,
when they are moving in opposite
directions.
Reason : Relative
velocity
of
A
w.r.t.

B vA vB
8.

Assertion

Vector addition of two vectors

A and B is commutative.

Reason : A B B A


9.
Assertion :
A.B B.A
Reason : Dot product of two vectors is
commutative.

10. Assertion :
r F and F r
Reason
commutative.

: Cross

product

of

vectors

is

Vectors
11.

Assertion

:
A negative acceleration of a
body is associated with a slowing
down of a body.
Reason : Acceleration is vector quantity.
12. Assertion :
A physical quantity cannot be
called as a vector if its magnitude is
zero.
Reason : A vector has both, magnitude and
direction.
13. Assertion :
The sum of two vectors can be
zero.
Reason : The vector cancel each other, when
they are equal and opposite.
14. Assertion :
Two vectors are said to be like
vectors if they have same direction
but different magnitude.
Reason : Vector quantities do not have specific
direction.
15. Assertion :
The scalar product of two
vectors can be zero.
Reason : If two vectors are perpendicular to
each other, their scalar product will be
zero.
16. Assertion :
Multiplying any vector by an
scalar is a meaningful operations.
Reason : In uniform motion speed remains
constant.
17. Assertion :
A null vector is a vector whose
magnitude is zero and direction is
arbitrary.
Reason : A null vector does not exist.
18. Assertion :
If dot product and cross product

of A and B are zero, it implies that

one of the vector A and B must be


a null vector.
Reason : Null vector is a vector with zero
magnitude.
19. Assertion :
The cross product of a vector
with itself is a null vector.
Reason : The cross-product of two vectors
results in a vector quantity.
20. Assertion :
The minimum number of non
coplanar vectors whose sum can be
zero, is four.
Reason : The resultant of two vectors of
unequal magnitude can be zero.

21. Assertion :
If A.B B.C, then A may

not always be equal to C


Reason : The dot product of two vectors
involves cosine of the angle between
the two vectors.
22. Assertion :
Vector addition is commutative.

Reason : (A B) (B A).

10

11

12

13

14

15

16

17

18

19

20

21

22

23

24

25

26

27

28

29

30

31

32

33

34

Addition and Subtraction of Vectors


1

10

11

12

13

14

15

16

17

18

19

20

21

22

23

24

25

26

27

28

29

30

31

32

33

34

35

36

37

38

39

40

41

42

43

44

45

46

47

48

49

50

51

52

53

Multiplication of Vectors
1

10

11

12

13

14

15

16

17

18

19

20

21

22

23

24

25

26

27

28

29

30

31

32

33

34

35

36

37

38

39

40

41

42

43

44

45

46

47

48

49

50

51

52

53

54

55

56

57

58

59

Lami's Theorem
1

Relative Velocity
1

10

ac

11

12

13

14

Critical Thinking Questions

Fundamentals of Vectors

10

11

12

13

14

Vectors

Assertion and Reason


1

10

11

12

13

14

15

16

17

18

19

20

21

22

You might also like